K
Khách

Hãy nhập câu hỏi của bạn vào đây, nếu là tài khoản VIP, bạn sẽ được ưu tiên trả lời.

18 tháng 7 2018

ta có : \(Q=C^1_n+2\dfrac{C_n^2}{C_n^1}+...+k\dfrac{C^k_n}{C_n^{k-1}}+...+n\dfrac{C^n_n}{C_n^{n-1}}\)

\(\Leftrightarrow Q=\dfrac{n!}{1!\left(n-1\right)!}+2\dfrac{1!\left(n-1\right)!}{2!\left(n-2\right)!}+...+k\dfrac{\left(k-1\right)!\left(n-k+1\right)!}{k!\left(n-k\right)!}+...+\dfrac{n\left(n-1\right)!1!}{n!}\)

\(\Leftrightarrow Q=n+\dfrac{2\left(n-1\right)}{2}+...+\dfrac{k\left(n-k+1\right)}{k}+...+\dfrac{n}{n}\)

\(\Leftrightarrow Q=n+\left(n-1\right)+...+\left(n-k+1\right)+...+1\)

\(\Leftrightarrow Q=n^2-\left(1+\left(1+1\right)+\left(1+2\right)+...+\left(n-1\right)\right)\)

AH
Akai Haruma
Giáo viên
8 tháng 7 2019

Lời giải:

Ta thực hiện chứng minh đẳng thức trên đúng bằng quy nạp

Với $n=2$: \((a+b)^=a^2+2ab+b^2=C^0_2a^2b^0+C^1_2ab+C^2_2a^0b^2\) (đúng)

................

Giả sử đẳng thức đúng đến $n=t$ $(t\in\mathbb{Z}>2$), tức là \((a+b)^t=\sum ^t_{k=0}C^k_ta^{t-k}b^k\)

Ta cần chứng minh nó cũng đúng với $n=t+1$. Thật vậy:

\((a+b)^{t+1}=(a+b)^t(a+b)=(a+b)\sum ^{t}_{k=0}a^{t-k}b^k\)

\(=C^0_ta^{t+1}+(C^1_t+C^0_t)a^tb+(C^2_t+C^1_t)a^{t-1}b^2+....+(C^t_t+C^{t-1}_t)ab^t+C^t_tb^{t+1}\)

\(=C^0_{t+1}a^{t+1}+C^1_{t+1}a^tb+C^2_{t+1}a^{t-1}b^2+....+C^t_{t+1}ab^t+C^{t+1}_{t+1}b^{t+1}\) (sử dụng đẳng thức \(C^k_n+C^{k+1}_n=C^{k+1}_{n+1}\)\(C^0_t=C^0_{t+1}=1; C^t_t=C^{t+1}_{t+1}=1\))

\(=\sum ^{t+1}_{k=0}C^{k}_{t+1}a^{t+1-k}b^k\)

Phép chứng minh hoàn tất. Ta có đpcm.

8 tháng 7 2019

chị Akai Haruma giúp em với

AH
Akai Haruma
Giáo viên
7 tháng 4 2023

Lời giải:

$1440=2^5.3^2.5$

Để $k=n!\vdots 1440$ thì $n!\vdots 2^5$; $n!\vdots 3^2; n!\vdots 5$

Để $n!\vdots 3^2; 5$ thì $n\geq 6(1)$

Để $n!\vdots 2^5$. Để ý $2=2^1, 4=2^2, 6=2.3, 8=2^3$. Để $n!\vdots 2^5$ thì $n\geq 8(2)$

Từ $(1); (2)$ suy ra $n\geq 8$. Giá tri nhỏ nhất của $n$ có thể là $8$

17 tháng 4 2022

D. k=\(\dfrac{1}{2}\)

17 tháng 4 2022

D.k=\(\dfrac{1}{2}\)

NV
17 tháng 4 2022

\(\overrightarrow{AC}+\overrightarrow{BD}=\overrightarrow{AM}+\overrightarrow{MN}+\overrightarrow{NC}+\overrightarrow{BM}+\overrightarrow{MN}+\overrightarrow{ND}\)

\(=2\overrightarrow{MN}+\left(\overrightarrow{AM}+\overrightarrow{BM}\right)+\left(\overrightarrow{NC}+\overrightarrow{ND}\right)\)

\(=2\overrightarrow{MN}\)

\(\Rightarrow k=\dfrac{1}{2}\)